Đến nội dung

Hoang Duong

Hoang Duong

Đăng ký: 10-05-2015
Offline Đăng nhập: 07-02-2017 - 21:30
-----

#655537 $(a+b)^{4}+(b+c)^{4}+(c+a)^{4}\geq...

Gửi bởi Hoang Duong trong 25-09-2016 - 20:05

Liệu có cách làm nào không dùng đến bất đẳng thức Holder không nhỉ  :)

Bài này điểm rơi Cauchy cũng chơi mượt mà được mà, chưa cần dùng đến Holder đâu,

$(a+b)^4+\frac{16}{81}+\frac{16}{81}+\frac{16}{81}\ge4\sqrt[4]{(a+b)(\frac{16}{81})^3}=\frac{32}{27}(a+b)$

Tương tự cộng lại ta có điều phải chứng minh  :D  :D  :D  :D  cần gì phải dao búa




#655517 Tìm MIN của $\sum \frac{a^{2}}{a^...

Gửi bởi Hoang Duong trong 25-09-2016 - 17:45

Dễ dàng chứng minh:

$(a+b+c+d+e)(\frac{1}{a}+\frac{1}{b}+\frac{1}{c}+\frac{1}{d}+\frac{1}{e})\ge25$

Hay:

$\frac{1}{\sum a}\le\sum\frac{1}{25}\sum\frac{1}{a}$

Quay lại bài toán:

$P=\sum\frac{a^2}{a^2+(b+c)^2}\ge\sum\frac{a^2}{a^2+2(b^2+c^2)}$

$\Rightarrow 3-P\le 2\sum\frac{b^2+c^2}{a^2+2(b^2+c^2)}=2\sum\frac{b^2+c^2}{\frac{3(a^2+b^2+c^2)}{3}+2\frac{(b^2+c^2)}{2}}$

$\le\frac{2}{25}(b^2+c^2)\sum(\frac{9}{a^2+b^2+c^2}+\frac{4}{b^2+c^2})$

$=\frac{2}{25}(9\sum\frac{b^2+c^2}{a^2+b^2+c^2}+12)=\frac{60}{25}=\frac{12}{5}$

Từ đó suy ra: $P\ge\frac{3}{5}$




#652742 CMR: $\sqrt{2a^{2}+ab+2b^{2}}+\s...

Gửi bởi Hoang Duong trong 04-09-2016 - 12:49

Dạng này: https://drive.google...b09WZUVsUmJUTDg




#647450 Cho $a,b,c\in [0;2]$ và $a+b+c=3$. Chứng minh rằng:...

Gửi bởi Hoang Duong trong 01-08-2016 - 08:39

$\sum ab\le\frac{(\sum a)^3}{3}=3$

Và:
$\prod(a-2)\le0 \Leftrightarrow \sum 2ab\ge abc+4\ge 4 \Rightarrow\sum ab\ge2$
Khai triển biểu thức:
$P=\sum a^3 -3\prod(a-1)=(\sum a)^3-3\sum a\sum ab+3abc-3(abc-\sum ab+\sum a-1)=21-6\sum ab$
Theo 2 đánh giá trên ta có ngay đpcm



#644361 $S=\left (\frac{a}{b+c} \right )^...

Gửi bởi Hoang Duong trong 10-07-2016 - 16:25

TH $k \geq 1 $ thì dễ dàng chứng minh được,
nhưng TH k<1 và a,b,c dương thì t chưa nghĩ ra




#644358 $S=\left (\frac{a}{b+c} \right )^...

Gửi bởi Hoang Duong trong 10-07-2016 - 16:13

Điều kiện của k chỉ mỗi vậy thôi à bạn




#643583 $x^{3}+3x^{2}\sqrt{x+1}-4(x+1)\sqrt{x+1}=0$

Gửi bởi Hoang Duong trong 04-07-2016 - 10:02

"Khó căng não" tức là dễ quá đúng không nhỉ




#638505 Tìm GTNN của P=1/x+4/y+9/z

Gửi bởi Hoang Duong trong 06-06-2016 - 14:44

Bài này C-S ra luôn, nhưng điểm rơi thì cũng làm được:

$\frac{1}{x}+a^2x\geq 2a$

$\frac{4}{y}+b^2y\geq 4b$

$\frac{9}{z}+c^2z\geq 6c$ (với a,b,c là các hằng số dương chọn sau

suy ra:

$\frac{1}{x}+\frac{4}{y}+\frac{9}{z}+b^2y+a^2x+c^2z\geq 2a+4b+6c$

dấu "=" xảy ra khi x=1/a,y=2/b,z=3/c

Cần chọn a,b,c sao cho $a^2=b^2=c^2$ hay a=b=cvà 1/a+2/c+3/c=1

vậy a=b=c=6
vậy là okê nhé bạn




#638147 $S=\dfrac{a}{1+a}+\dfrac{3b}...

Gửi bởi Hoang Duong trong 04-06-2016 - 23:42

thế vào rồi khảo sát hàm thôi chứ dồn làm chi cho mất công cậu




#637995 Tổng hợp các bài BĐT trong các đề thi thử THPT Quốc Gia môn Toán năm 2017

Gửi bởi Hoang Duong trong 04-06-2016 - 13:11

Bài 74: Cho x,y,x là các số thực dương thỏa mãn: $x^2+y^2+z^2=1$. Tìm GTNN của biểu thức:

$\sum \frac{x}{1-x^2}$

Ta có bđt sau: $\frac{1}{1-x^2}\geq\frac{3\sqrt{3}x}{2}\Leftrightarrow (x-\frac{1}{\sqrt3})^2(3\sqrt3x+6)\geq 0$ (luôn đúng)

suy ra:

$\sum\frac{x}{1-x^2}\geq\sum\frac{3\sqrt3x^2}{2}=\frac{3\sqrt3}{2}$
"=" xảy ra khi $x=y=z=\frac{1}{\sqrt3}$




#632317 Tìm tọa độ của E

Gửi bởi Hoang Duong trong 10-05-2016 - 20:35

Đây là câu 9 trong đề thi thử chuyên Vinh lần 3,
bạn hãy chứng minh tg HEK vuông và AH=HE là okê, kêt quả có 2 điểm A nhé bạn :)




#625819 $a^{m+n}+b^{m+n}+c^{m+n}\geq a^{...

Gửi bởi Hoang Duong trong 08-04-2016 - 05:55

$ma^{m+n}+nb^{m+n}\geq (m+n)a^{m}b^{n}$

thiết lập tương tự cộng lại có đpcm




#619673 Tiếp sức bất đẳng thức

Gửi bởi Hoang Duong trong 11-03-2016 - 13:25

đổi biến (a,b,c)~($\frac{2x}{y},\frac{2y}{z},\frac{2z}{x}$)
=>$ P= \sum \frac{yz}{4xz+2y^2+6yz} $
$3-6P=\sum \frac{4xz+2y^2}{4xz+2y^2+6yz}$

ta có $\sum \frac{4xz}{4xz+6yz+2y^2} \geq \frac{4(xy+xz+yz)^2}{4((xz)^2+(yz)^2+(xy)^2) +8xyz (x+y+z)} =1 $

và     $ \sum \frac{2y^2}{4xz+2y^2+6xz} \geq \frac{2(x+z+z)^2}{2(x^2+y^2+z^2)+10(xz+yz+xy)} = \frac{(x+y+z)^2}{(x+y+z)^2+3(xz+yz+xy)} \geq \frac{(x+y+z)^2}{(x+y+z)^2+3*\frac{(x+y+z)^2}{3}} =\frac{1}{1+1}=\frac{1}{2}$
Vậy $3-6P \geq \frac{1}{2}+1$ => $P \leq \frac{1}{4}$

Lời giải khá hay
nhưng có thể hơi khó hiểu,

mình xin trình bày lời giải khác:

đổi biến: $(x,y,z)\rightarrow (2a^2,2b^2,2c^2)\Rightarrow abc=1$

khi đó bất đẳng thức trở thành:

$\sum\frac{1}{4a^2+2b^2+6}=\sum\frac{1}{2a^2+2b^2+2a^2+2+4}\leq\sum\frac{1}{4ab+4a+4}=\frac{1}{4}\sum\frac{1}{ab+a+1}=1/4$

Công việc chứng minh $\sum\frac{1}{ab+a+1}=1$ với $abc=1$ khá đơn giản, bạn chỉ cần rút thế hợp lí là được




#619672 Tiếp sức bất đẳng thức

Gửi bởi Hoang Duong trong 11-03-2016 - 13:09

Cho a,b,c>0 C/m: $\frac{a^2}{1+b-a}+\frac{b^2}{1+c-b}+\frac{c^2}{1+a-c}\geqslant 1$

Mình chắc là phải có giả thiết $a^2+b^2+c^2=1$ hoặc tương tự thì mới ổn
ta có:

$\sum c^3=\sum\frac{a^3+a^3+b^3}{3}\geq \sum a^2b$

Áp dụng:

$\sum\frac{a^2}{1+b-a}=\frac{a^4}{a^2+a^2b-a^3}\geq\sum\frac{(\sum a^2)}{\sum a^2+\sum a^2b-\sum a^3}\geq\sum a^2=1$




#618973 $\sum \sqrt{\frac{a^{3}}{a^...

Gửi bởi Hoang Duong trong 07-03-2016 - 21:22

Ta có: $\sqrt{1+x^3}=\sqrt{(1+x)(1-x+x^{2})}\leq\frac{(1+x)+(1-x+x2)}{2}=\frac{2+x^{2}}{2}. \Rightarrow \frac{1}{\sqrt{1+x^{3}}}\geq\frac{2}{2+x^{3}}$

ÁP dụng:

$\sqrt{\frac{a^{3}}{a^{3}+(b+c)^{3}}}=\frac{1}{\sqrt{1+(\frac{c+b}{a})^{3}}}\geq\frac{2}{1+(\frac{c+b}{a})^{2}}=\frac{2a^{2}}{2a^{2}+(b+c)^{2}}\geq\frac{2a^{2}}{2a^{2}+2(b^{2}+b^{2})}$

thiết lập tương tự cộng lại ta có đpcm